A Generalisation on an RMO Problem

ak(ab)(ac)+bk(ba)(bc)+ck(ca)(cb)>k(k1)2\dfrac {a^k}{(a-b)(a-c)} + \dfrac {b^k}{(b-a)(b-c)} + \dfrac {c^k}{(c-a)(c-b)} > \dfrac {k(k-1)}{2}

If aa, bb and cc are distinct positive reals such that abc=1abc = 1, and k3k \geq 3 is a positive integer, show that the inequality above holds.

Note: The RMO problem had the case k=3k=3.

#Algebra

Note by Sharky Kesa
4 years, 8 months ago

No vote yet
1 vote

  Easy Math Editor

This discussion board is a place to discuss our Daily Challenges and the math and science related to those challenges. Explanations are more than just a solution — they should explain the steps and thinking strategies that you used to obtain the solution. Comments should further the discussion of math and science.

When posting on Brilliant:

  • Use the emojis to react to an explanation, whether you're congratulating a job well done , or just really confused .
  • Ask specific questions about the challenge or the steps in somebody's explanation. Well-posed questions can add a lot to the discussion, but posting "I don't understand!" doesn't help anyone.
  • Try to contribute something new to the discussion, whether it is an extension, generalization or other idea related to the challenge.
  • Stay on topic — we're all here to learn more about math and science, not to hear about your favorite get-rich-quick scheme or current world events.

MarkdownAppears as
*italics* or _italics_ italics
**bold** or __bold__ bold

- bulleted
- list

  • bulleted
  • list

1. numbered
2. list

  1. numbered
  2. list
Note: you must add a full line of space before and after lists for them to show up correctly
paragraph 1

paragraph 2

paragraph 1

paragraph 2

[example link](https://brilliant.org)example link
> This is a quote
This is a quote
    # I indented these lines
    # 4 spaces, and now they show
    # up as a code block.

    print "hello world"
# I indented these lines
# 4 spaces, and now they show
# up as a code block.

print "hello world"
MathAppears as
Remember to wrap math in \( ... \) or \[ ... \] to ensure proper formatting.
2 \times 3 2×3 2 \times 3
2^{34} 234 2^{34}
a_{i-1} ai1 a_{i-1}
\frac{2}{3} 23 \frac{2}{3}
\sqrt{2} 2 \sqrt{2}
\sum_{i=1}^3 i=13 \sum_{i=1}^3
\sin \theta sinθ \sin \theta
\boxed{123} 123 \boxed{123}

Comments

Anyone got a better solution than this?

Firstly, we add the fractions together (using the fact ba=(ab)b-a=-(a-b), etc. to get

ak(ab)(ac)+bk(ba)(bc)+ck(ca)(cb)=ak(bc)bk(ac)+ck(ab)(ab)(ac)(bc)=akbabkakc+bkc+ck(ab)(ab)(ac)(bc)=ab(ab)(ak2++bk2)c(ab)(ak1++bk1)+ck(ab)(ab)(ac)(bc)=ak1b++abk1ak1c++bk1c+ck(ac)(bc)=ak1bak1c+ak2b2ak2bc++abk1abk2cbk1c+ck(ac)(bc)=(bc)(ak1+ak2b++abk2)c(bc)(bk2++ck2)(ac)(bc)=ak1++abk2bk2cck1ac=ak1ck1+ak2bbck2++abk2bk2cac=(ac)(ak2++ck2)+b(ac)(ak3++ck3)++bk2(ac)ac=ak2++ck2+ak3b++bck3++bk2\begin{aligned} \dfrac {a^k}{(a-b)(a-c)} + \dfrac {b^k}{(b-a)(b-c)} + \dfrac {c^k}{(c-a)(c-b)} &= \dfrac{a^k (b-c) - b^k (a-c) + c^k (a-b)}{(a-b)(a-c)(b-c)}\\ &= \dfrac {a^k b - ab^k - a^k c + b^k c + c^k (a-b)}{(a-b)(a-c)(b-c)}\\ &= \dfrac {ab (a-b)(a^{k-2} + \ldots + b^{k-2}) - c (a-b)(a^{k-1} + \ldots + b^{k-1}) + c^k (a-b)}{(a-b)(a-c)(b-c)}\\ &= \dfrac {a^{k-1} b + \ldots + ab^{k-1} - a^{k-1} c + \ldots + b^{k-1} c + c^k}{(a-c)(b-c)}\\ &= \dfrac {a^{k-1} b - a^{k-1} c + a^{k-2} b^2 - a^{k-2} bc + \ldots + ab^{k-1} - ab^{k-2} c - b^{k-1} c + c^k}{(a-c)(b-c)}\\ &= \dfrac {(b-c)(a^{k-1} + a^{k-2}b + \ldots + ab^{k-2}) - c(b-c)(b^{k-2} + \ldots + c^{k-2})}{(a-c)(b-c)}\\ &= \dfrac {a^{k-1} + \ldots + ab^{k-2} - b^{k-2}c - \ldots - c^{k-1}}{a-c}\\ &= \dfrac {a^{k-1} - c^{k-1} + a^{k-2} b - b c^{k-2} + \ldots + ab^{k-2} - b^{k-2} c}{a-c}\\ &= \dfrac {(a-c)(a^{k-2} + \ldots + c^{k-2}) + b(a-c)(a^{k-3} + \ldots + c^{k-3}) + \ldots + b^{k-2} (a-c)}{a-c}\\ &= a^{k-2} + \ldots + c^{k-2} + a^{k-3} b + \ldots + b c^{k-3} + \ldots + b^{k-2} \end{aligned}

Notice that this expression goes through every single possible term composed of variables a,b,ca, b, c and degree k2k-2. Thus, by Supermarket Principle, there are (k2)\binom{k}{2} terms here. Notice that the product of all the terms will have equal degrees for each of aa, bb and cc by symmetry, so their product will be just 1 (using abc=1abc = 1). Also, we must have aa, bb and cc distinct for the original expression to have a defined value. Thus, by AM-GM, we have

ak2++ck2+ak3b++bck3++bk2(k2)>1ak2++ck2+ak3b++bck3++bk2>(k2)=k(k1)2\begin{aligned} \dfrac {a^{k-2} + \ldots + c^{k-2} + a^{k-3} b + \ldots + b c^{k-3} + \ldots + b^{k-2}}{\dbinom{k}{2}} &> 1\\ a^{k-2} + \ldots + c^{k-2} + a^{k-3} b + \ldots + b c^{k-3} + \ldots + b^{k-2} &> \dbinom {k}{2} = \dfrac {k(k-1)}{2} \end{aligned}

Thus, proven.

Sharky Kesa - 4 years, 7 months ago

Sharky, you've got the right idea. There is a much easier way to show that identity

Claim: am(bc)+bm(ca)+cm(ab)=(a2ba2c+b2cb2a+c2ac2b)i+j+k=m2aibjck a^m (b-c) + b^m(c-a) + c^m (a-b) = (a^2b-a^2c+b^2c-b^2a+c^2a-c^2b) \sum_{i + j + k = m-2} a^i b^j c^k

Proof: Let's find the coefficient of the term apbqcr, a^p b^q c^r, where p+q+r=m+1, p+q+r = m+1, on the RHS.

  • If 2p,q,r 2 \leq p, q, r , then the coefficient would be 0, because we can obtain it via multiplying through all 6 terms in the first factor.
  • If {p,q,r}={0,0,m+1} \{p,q,r\} = \{0,0, m+1\} , then the coefficient would be 0 because we are multiplying by cross terms in the first factor.
  • If {p,q,r}={0,1,m} \{p,q,r\} = \{ 0, 1, m \} , show that it has the desired coefficient.
  • If {p,q,r}={1,1,m1} \{ p, q, r \} = \{ 1, 1, m-1 \} , show that it has the desired coefficient of 0.

Sometimes if you know what you want, it's best to prove it directly instead of trying to explain how you found it out. Esp if the way that you first found it provides no additional insight into the process.

Calvin Lin Staff - 4 years, 7 months ago

@Svatejas Shivakumar To be clear, your approach works for general nn through a simple generalization.

Calvin Lin Staff - 4 years, 7 months ago
×

Problem Loading...

Note Loading...

Set Loading...